K
Khách

Hãy nhập câu hỏi của bạn vào đây, nếu là tài khoản VIP, bạn sẽ được ưu tiên trả lời.

AH
Akai Haruma
Giáo viên
19 tháng 12 2021

Lời giải:

Dấu "=" không xảy ra.
Áp dụng BĐT AM-GM:

\(\text{VT}\leq \frac{a+(b+1)}{2}+\frac{b+(c+1)}{2}+\frac{c+(a+1)}{2}=\frac{2(a+b+c)+3}{2}\)

\(< \frac{3(a+b+c+ab+bc+ac+abc+1)}{2}=\frac{3(a+1)(b+1)(c+1)}{2}\)

Ta có đpcm.

AH
Akai Haruma
Giáo viên
19 tháng 12 2021

Lần sau bạn lưu ý đăng 1 bài 1 lần thôi. Đăng nhiều lần coi như spam và sẽ bị xóa không thương tiếc đấy nhé.

19 tháng 12 2021

ai giỏi ạ

19 tháng 12 2021

mk lớp 7

19 tháng 12 2021

Dấu '' = '' không xảy ra

Áp dụng BĐT AM-GM:

Dấu "=" không xảy ra.
Áp dụng BĐT AM-GM:

\(\text{VT}\leq \frac{a+(b+1)}{2}+\frac{b+(c+1)}{2}+\frac{c+(a+1)}{2}=\frac{2(a+b+c)+3}{2}\)

\(< \frac{3(a+b+c+ab+bc+ac+abc+1)}{2}=\frac{3(a+1)(b+1)(c+1)}{2}\)

Ta có đpcm.

28 tháng 8 2020

Áp dụng giả thiết và một đánh giá quen thuộc, ta được: \(16\left(a+b+c\right)\ge\frac{1}{a}+\frac{1}{b}+\frac{1}{c}=\frac{ab+bc+ca}{abc}=\frac{\left(ab+bc+ca\right)^2}{abc\left(ab+bc+ca\right)}\ge\frac{3\left(a+b+c\right)}{ab+bc+ca}\)hay \(\frac{1}{6\left(ab+bc+ca\right)}\le\frac{8}{9}\)

Đến đây, ta cần chứng minh \(\frac{1}{\left(a+b+\sqrt{2\left(a+c\right)}\right)^3}+\frac{1}{\left(b+c+\sqrt{2\left(b+a\right)}\right)^3}+\frac{1}{\left(c+a+\sqrt{2\left(c+b\right)}\right)^3}\le\frac{1}{6\left(ab+bc+ca\right)}\)

 Áp dụng bất đẳng thức Cauchy cho ba số dương ta có \(a+b+\sqrt{2\left(a+c\right)}=a+b+\sqrt{\frac{a+c}{2}}+\sqrt{\frac{a+c}{2}}\ge3\sqrt[3]{\frac{\left(a+b\right)\left(a+c\right)}{2}}\)hay \(\left(a+b+\sqrt{2\left(a+c\right)}\right)^3\ge\frac{27\left(a+b\right)\left(a+c\right)}{2}\Leftrightarrow\frac{1}{\left(a+b+2\sqrt{a+c}\right)^3}\le\frac{2}{27\left(a+b\right)\left(a+c\right)}\)

Hoàn toàn tương tự ta có \(\frac{1}{\left(b+c+2\sqrt{b+a}\right)^3}\le\frac{2}{27\left(b+c\right)\left(b+a\right)}\)\(\frac{1}{\left(c+a+2\sqrt{c+b}\right)^3}\le\frac{2}{27\left(c+a\right)\left(c+b\right)}\)

Cộng theo vế các bất đẳng thức trên ta được \(\frac{1}{\left(a+b+\sqrt{2\left(a+c\right)}\right)^3}+\frac{1}{\left(b+c+\sqrt{2\left(b+a\right)}\right)^3}+\frac{1}{\left(c+a+\sqrt{2\left(c+b\right)}\right)^3}\le\frac{4\left(a+b+c\right)}{27\left(a+b\right)\left(b+c\right)\left(c+a\right)}\)Phép chứng minh sẽ hoàn tất nếu ta chỉ ra được \(\frac{4\left(a+b+c\right)}{27\left(a+b\right)\left(b+c\right)\left(c+a\right)}\le\frac{1}{6\left(ab+bc+ca\right)}\)\(\Leftrightarrow\left(a+b\right)\left(b+c\right)\left(c+a\right)\ge\frac{8}{9}\left(ab+bc+ca\right)\left(a+b+c\right)\)

Đây là một đánh giá đúng, thật vậy: đặt a + b + c = p; ab + bc + ca = q; abc = r thì bất đẳng thức trên trở thành \(pq-r\ge\frac{8}{9}pq\Leftrightarrow\frac{1}{9}pq\ge r\)*đúng vì \(a+b+c\ge3\sqrt[3]{abc}\)\(ab+bc+ca\ge3\sqrt[3]{\left(abc\right)^2}\))

Vậy bất đẳng thức được chứng minh

Đẳng thức xảy ra khi \(a=b=c=\frac{1}{4}\)

2 tháng 12 2020

Đặt \(\left(\frac{1}{a},\frac{1}{b},\frac{1}{c}\right)=\left(x,y,z\right)\)

\(x+y+z\ge\frac{x^2+2xy}{2x+y}+\frac{y^2+2yz}{2y+z}+\frac{z^2+2zx}{2z+x}\)

\(\Leftrightarrow x+y+z\ge\frac{3xy}{2x+y}+\frac{3yz}{2y+z}+\frac{3zx}{2z+x}\)

\(\frac{3xy}{2x+y}\le\frac{3}{9}xy\left(\frac{1}{x}+\frac{1}{x}+\frac{1}{y}\right)=\frac{1}{3}\left(x+2y\right)\)

\(\Rightarrow\Sigma_{cyc}\frac{3xy}{2x+y}\le\frac{1}{3}\left[\left(x+2y\right)+\left(y+2z\right)+\left(z+2x\right)\right]=x+y+z\)

Dấu "=" xảy ra khi x=y=z

25 tháng 12 2020

Áp dụng bất đẳng thức Cauchy-Schwarz ta có:

\(\sqrt{\left(a+b\right)\left(a+c\right)}\ge\sqrt{a}.\sqrt{a}+\sqrt{b}.\sqrt{c}\)

\(\Leftrightarrow\sqrt{\left(a+b\right)\left(a+c\right)}\ge a+\sqrt{bc}\)

Do đó \(\sqrt{\frac{bc}{\left(c+a\right)\left(a+b\right)}}=\frac{\sqrt{bc\left(c+a\right)\left(a+b\right)}}{\left(c+a\right)\left(a+b\right)}\ge\sqrt{abc}\frac{\sqrt{a}}{\left(c+a\right)\left(c+b\right)}+\frac{bc}{\left(c+a\right)\left(c+b\right)}\left(1\right)\)

Chứng minh tương tự ta được: 

\(\hept{\begin{cases}\sqrt{\frac{bc}{\left(c+b\right)\left(a+b\right)}}=\frac{\sqrt{bc\left(c+b\right)\left(a+b\right)}}{\left(c+b\right)\left(a+b\right)}\ge\sqrt{abc}\frac{\sqrt{b}}{\left(c+b\right)\left(a+b\right)}+\frac{ac}{\left(c+b\right)\left(a+b\right)}\left(2\right)\\\sqrt{\frac{ca}{\left(c+a\right)\left(a+b\right)}}=\frac{\sqrt{ca\left(c+a\right)\left(a+b\right)}}{\left(c+a\right)\left(a+b\right)}\ge\sqrt{abc}\frac{\sqrt{c}}{\left(c+a\right)\left(a+b\right)}+\frac{ab}{\left(a+c\right)\left(a+b\right)}\left(3\right)\end{cases}}\)

\(\Rightarrow\sqrt{\frac{bc}{\left(c+a\right)\left(a+b\right)}}+\sqrt{\frac{ca}{\left(c+b\right)\left(a+b\right)}}+\sqrt{\frac{ab}{\left(a+c\right)\left(b+c\right)}}\ge\)

\(\sqrt{abc}\left(\frac{\sqrt{a}}{\left(a+c\right)\left(a+b\right)}+\frac{\sqrt{b}}{\left(c+b\right)\left(a+b\right)}+\frac{\sqrt{c}}{\left(c+b\right)\left(a+c\right)}\right)+\)\(\frac{bc}{\left(a+c\right)\left(a+b\right)}+\frac{ac}{\left(c+b\right)\left(a+b\right)}+\frac{ab}{\left(c+b\right)\left(a+c\right)}\left(4\right)\)

Ta lại có: \(\frac{bc}{\left(a+c\right)\left(a+b\right)}+\frac{ac}{\left(c+b\right)\left(a+b\right)}+\frac{ab}{\left(c+b\right)\left(a+c\right)}+\frac{2abc}{\left(a+b\right)\left(b+c\right)\left(c+a\right)}\)

\(=\frac{bc\left(b+c\right)+ac\left(a+c\right)+ab\left(a+b\right)+2abc}{\left(a+c\right)\left(b+c\right)\left(a+b\right)}\)

\(=\frac{bc\left(a+b+c\right)+ca\left(a+b+c\right)+ab\left(a+b\right)}{\left(a+b\right)\left(b+c\right)\left(c+a\right)}=\frac{c\left(a+b+c\right)\left(b+a\right)+ab\left(a+b\right)}{\left(a+b\right)\left(b+c\right)\left(c+a\right)}\)

\(=\frac{\left(a+b\right)\left[c\left(a+c\right)+b\left(a+c\right)\right]}{\left(a+b\right)\left(b+c\right)\left(c+a\right)}=\frac{\left(a+b\right)\left(c+b\right)\left(a+c\right)}{\left(a+b\right)\left(b+c\right)\left(c+a\right)}=1\)

\(\left(4\right)\Leftrightarrow\sqrt{\frac{bc}{\left(c+a\right)\left(a+b\right)}}+\sqrt{\frac{ca}{\left(c+b\right)\left(a+b\right)}}+\sqrt{\frac{ab}{\left(a+c\right)\left(b+c\right)}}\)\(\ge\sqrt{abc}\left(\frac{\sqrt{a}}{\left(c+a\right)\left(a+b\right)}+\frac{\sqrt{b}}{\left(c+b\right)\left(a+b\right)}+\frac{\sqrt{c}}{\left(c+b\right)\left(a+c\right)}\right)+1-\frac{2abc}{\left(a+b\right)\left(b+c\right)\left(c+a\right)}\)

Do đó ta cần chứng minh \(\sqrt{abc}\left(\frac{\sqrt{a}}{\left(c+a\right)\left(a+b\right)}+\frac{\sqrt{b}}{\left(c+b\right)\left(a+b\right)}+\frac{\sqrt{c}}{\left(c+b\right)\left(a+c\right)}\right)+1-\frac{2abc}{\left(a+b\right)\left(b+c\right)\left(c+a\right)}\)\(\ge1+\frac{4abc}{\left(a+b\right)\left(b+c\right)\left(c+a\right)}\)

Điều này tương đương với \(\sqrt{a}\left(b+c\right)+\sqrt{b}\left(a+c\right)+\sqrt{c}\left(a+b\right)\ge6\sqrt{abc}\left(5\right)\)

Theo bất đẳng thức AM-GM thì (5) luôn đúng

Dấu "=" xảy ra khi (1);(2);(3) và (5) xảy ra dấu "=". điều này tương đương với a=b=c

Vậy ta có điều phải chứng minh

=))

25 tháng 6 2017

ko cả biết BĐT AM-GM với C-S là gì còn hỏi bài này rảnh háng

26 tháng 6 2017

Đề sai rồi. Nếu như là a, b, c dương thì giá trị nhỏ nhất của nó phải là 9 mới đúng. Còn để có GTNN như trên thì điều kiện là a, b, c không âm nhé. Mà bỏ đi e thi cái gì mà phải giải câu cỡ này. Cậu này mạnh lắm đấy không phải dạng thường đâu.

28 tháng 12 2020

Đề sai. Nếu chỗ căn vế phải mà là căn bậc 3 thì t sol cho